Prove sequence converges to sup

  • Thread starter Thread starter CAF123
  • Start date Start date
  • Tags Tags
    Sequence
Click For Summary
A nonempty bounded set E in R has a supremum sup E that is not in E. To prove the existence of a strictly increasing sequence {x_n} in E converging to sup E, one can use the approximation property of suprema. For any ε > 0, there exists an x in E such that sup E - ε < x, allowing the construction of a sequence x_n where x_n > sup E - 1/n. This sequence is strictly increasing and converges to sup E, confirming the required properties. The proof hinges on the completeness of R and the properties of suprema.
CAF123
Gold Member
Messages
2,918
Reaction score
87

Homework Statement


Suppose that E is contained in ##\mathbb{R}## is a nonempty bounded set and that ##\sup E## is not in E. Prove that there exists a strictly increasing sequence ##\left\{x_n\right\}## that converges to ##\sup E## such that ##x_n \in E## for all n in ##\mathbb{N}##.

Homework Equations


More like Relevant theorems:
Bolzano-Weirstrass,
Completeness,
Monotone convergence.

The Attempt at a Solution



E is contained in R and nonempty so by Completeness, sup E exists. E is bounded so -a < e < a for all e in E and let supE = a which is not in E. I suppose this might be the essence of the proof, but how exactly do we know such a sequence in E exists? If there existed a function ##f: \mathbb{N} \rightarrow \mathbb{R}## with all the ##x_n## in E, then provided I can show such a sequence is strictly increasing (and that it exists) then the rest of the problem becomes trivial. I thought about using nested intervals but I didn't get very far. Any ideas?

Many thanks.
 
Physics news on Phys.org
Here's an outline/suggestion on how to proceed: let sup(E)=s_E.
1) Can you show that given any \epsilon&gt;0, there exists an x\in E such that s_E-\epsilon&lt;x?
2) Consider the sequence \epsilon_n=\frac{1}{n}; apply this sequence of epsilons to part 1 to show that you can generate a sequence in E, call it x_n.
3) Show that this sequence converges to sup(E)=s_E.

Hint for part 1): this is a good exercise in proof by contradiction which teaches you how to negate quantifiers; "for all"'s become "there exists" and vice-versa, and statements get negated.
 
christoff said:
Here's an outline/suggestion on how to proceed: let sup(E)=s_E.
1) Can you show that given any \epsilon&gt;0, there exists an x\in E such that s_E-\epsilon&lt;x?
Yes, this is the Approximation property for suprema

2) Consider the sequence \epsilon_n=\frac{1}{n}; apply this sequence of epsilons to part 1 to show that you can generate a sequence in E, call it x_n.
I can make the following statements: supE - 1 < x for some x, supE - 1/2 < x for some x, etc.. So supE-1, supE-1/2,...supE-1/n.. would make a sequence and indeed it would be strictly increasing. My only worry is that how can I be sure that the elements would be in E?
 
CAF123 said:
Yes, this is the Approximation property for supremaI can make the following statements: supE - 1 < x for some x, supE - 1/2 < x for some x, etc.. So supE-1, supE-1/2,...supE-1/n.. would make a sequence and indeed it would be strictly increasing. My only worry is that how can I be sure that the elements would be in E?

The numbers Sup(E)-1, Sup(E)-1/2, Sup(E)-1/3,... aren't the numbers you're interested in; it's the x's that bound them that you want to look at.

Let's start at n=1. You said that there exists an x\in E such that Sup(E)-1&lt;x. Let's call this x we found x_1.
Now, n=2. We know that there exists an x\in S such that Sup(E)-1/2&lt;x. Let's call this second x we've found x_2.
Then we could do the same for n=3, n=4, etc.

The important part thing is we can do this for ANY n; so in particular, there exists a sequence x_n\in E with the property that Sup(E)-1/n&lt;x_n. Can you show that this sequence converges? Hint below if you can't get it.

You also know that, because Sup(E) is the supremum, that each x_n is bounded by Sup(E). So you have Sup(E)-1/n&lt;x_n\leq Sup(E).
 
Last edited:
christoff said:
The numbers Sup(E)-1, Sup(E)-1/2, Sup(E)-1/3,... aren't the numbers you're interested in; it's the x's that bound them that you want to look at.

Let's start at n=1. You said that there exists an x\in E such that Sup(E)-1&lt;x. Let's call this x we found x_1.
Now, n=2. We know that there exists an x\in S such that Sup(E)-1/2&lt;x. Let's call this second x we've found x_2.
Then we could do the same for n=3, n=4, etc.

The important part thing is we can do this for ANY n; so in particular, there exists a sequence x_n\in E with the property that Sup(E)-1/n&lt;x_n. Can you show that this sequence converges? Hint below if you can't get it.

You also know that, because Sup(E) is the supremum, that each x_n is bounded by Sup(E). So you have Sup(E)-1/n&lt;x_n\leq Sup(E).

Ok so the last statement implies $$\lim_{n \rightarrow \infty} \sup E - \lim_{n \rightarrow \infty} \frac{1}{n} \leq \lim_{n \rightarrow \infty} x_n$$ From this ##x_n## must converge to ##\sup E##. It caanot be greater because the ##x_n## are bounded above by ##\sup E##. Is this all I need to do? I haven't checked your hint yet, I prefer to have feedback first.
 
CAF123 said:
Ok so the last statement implies $$\lim_{n \rightarrow \infty} \sup E - \lim_{n \rightarrow \infty} \frac{1}{n} \leq \lim_{n \rightarrow \infty} x_n$$ From this ##x_n## must converge to ##\sup E##. It caanot be greater because the ##x_n## are bounded above by ##\sup E##. Is this all I need to do? I haven't checked your hint yet, I prefer to have feedback first.

Yes, that is basically the idea. Since the sequence sup(E)-1/n converges to sup(E), and x_n is bounded above by the sup(R) and below by this sup(E)-1/n, it must converge to sup(E).
 
Question: A clock's minute hand has length 4 and its hour hand has length 3. What is the distance between the tips at the moment when it is increasing most rapidly?(Putnam Exam Question) Answer: Making assumption that both the hands moves at constant angular velocities, the answer is ## \sqrt{7} .## But don't you think this assumption is somewhat doubtful and wrong?

Similar threads

  • · Replies 11 ·
Replies
11
Views
3K
Replies
4
Views
2K
  • · Replies 13 ·
Replies
13
Views
2K
  • · Replies 13 ·
Replies
13
Views
2K
  • · Replies 14 ·
Replies
14
Views
3K
  • · Replies 8 ·
Replies
8
Views
3K
Replies
15
Views
2K
  • · Replies 1 ·
Replies
1
Views
2K
Replies
3
Views
2K
Replies
6
Views
3K